Showing given function is real analytic on R












0














I was reading about real analytic functions and I understood that to show a function is real analytic on $mathbb{R}$, we need to show that there is a power series centered at $x_0$ which converges to an open interval around $x_0$ $forall x_0 in mathbb{R}$


The example I am working on is the function $$f(x) = frac{1}{1+x^2}$$
I can show that it converges for expansion around 0 by expanding it as a geometric series and this converges in (-1,1) when centered at x=0. How do I show it for other x $in mathbb{R}$ ?


I think I remember a fact that if power series centered around x=0 converges in (-1,1) then power series centered at any point in this interval would also converge(can someone give verify if this is indeed true?). But that still leaves us with the rest of the number line.










share|cite|improve this question






















  • Have you learned about Taylor Series?
    – JavaMan
    Nov 21 '18 at 5:38










  • Yes, I have. Yea, maybe we can use taylor series to find the expansion. How do we show it converges? Also, is my statement correct about convergence about a different point in the interval?
    – Red Floyd
    Nov 21 '18 at 6:22






  • 1




    To show a Taylor Series converges, you have to show that $R_n(x) = sum_{k=n+1}^{infty} frac{f^{(k)}(a) (x-a)^k}{k!} to 0$. By Lagrange's Thm $R_n(x) = frac{f^{(n+1)}(c) (x-a)^{n+1}}{(n+1)!}$ for some $c$ between $a$ and $x$. Now, the $n$th derivative is something like $f(x) leq C x^{-n-2}$. Hence, $f^{(n)}(x) to 0$ as $n to infty$. In particular $f^{(n+1)}(c)$ is bounded. Finally, $(x-a)^{n+1}$ is polynomial, and factorials go to zero faster than any polynomial (this follows since $x^n/n! to 0$ as $n to infty$), which is demonstrated by convergence of Taylor series for $e^x$.
    – JavaMan
    Nov 22 '18 at 6:21
















0














I was reading about real analytic functions and I understood that to show a function is real analytic on $mathbb{R}$, we need to show that there is a power series centered at $x_0$ which converges to an open interval around $x_0$ $forall x_0 in mathbb{R}$


The example I am working on is the function $$f(x) = frac{1}{1+x^2}$$
I can show that it converges for expansion around 0 by expanding it as a geometric series and this converges in (-1,1) when centered at x=0. How do I show it for other x $in mathbb{R}$ ?


I think I remember a fact that if power series centered around x=0 converges in (-1,1) then power series centered at any point in this interval would also converge(can someone give verify if this is indeed true?). But that still leaves us with the rest of the number line.










share|cite|improve this question






















  • Have you learned about Taylor Series?
    – JavaMan
    Nov 21 '18 at 5:38










  • Yes, I have. Yea, maybe we can use taylor series to find the expansion. How do we show it converges? Also, is my statement correct about convergence about a different point in the interval?
    – Red Floyd
    Nov 21 '18 at 6:22






  • 1




    To show a Taylor Series converges, you have to show that $R_n(x) = sum_{k=n+1}^{infty} frac{f^{(k)}(a) (x-a)^k}{k!} to 0$. By Lagrange's Thm $R_n(x) = frac{f^{(n+1)}(c) (x-a)^{n+1}}{(n+1)!}$ for some $c$ between $a$ and $x$. Now, the $n$th derivative is something like $f(x) leq C x^{-n-2}$. Hence, $f^{(n)}(x) to 0$ as $n to infty$. In particular $f^{(n+1)}(c)$ is bounded. Finally, $(x-a)^{n+1}$ is polynomial, and factorials go to zero faster than any polynomial (this follows since $x^n/n! to 0$ as $n to infty$), which is demonstrated by convergence of Taylor series for $e^x$.
    – JavaMan
    Nov 22 '18 at 6:21














0












0








0







I was reading about real analytic functions and I understood that to show a function is real analytic on $mathbb{R}$, we need to show that there is a power series centered at $x_0$ which converges to an open interval around $x_0$ $forall x_0 in mathbb{R}$


The example I am working on is the function $$f(x) = frac{1}{1+x^2}$$
I can show that it converges for expansion around 0 by expanding it as a geometric series and this converges in (-1,1) when centered at x=0. How do I show it for other x $in mathbb{R}$ ?


I think I remember a fact that if power series centered around x=0 converges in (-1,1) then power series centered at any point in this interval would also converge(can someone give verify if this is indeed true?). But that still leaves us with the rest of the number line.










share|cite|improve this question













I was reading about real analytic functions and I understood that to show a function is real analytic on $mathbb{R}$, we need to show that there is a power series centered at $x_0$ which converges to an open interval around $x_0$ $forall x_0 in mathbb{R}$


The example I am working on is the function $$f(x) = frac{1}{1+x^2}$$
I can show that it converges for expansion around 0 by expanding it as a geometric series and this converges in (-1,1) when centered at x=0. How do I show it for other x $in mathbb{R}$ ?


I think I remember a fact that if power series centered around x=0 converges in (-1,1) then power series centered at any point in this interval would also converge(can someone give verify if this is indeed true?). But that still leaves us with the rest of the number line.







real-analysis analysis convergence






share|cite|improve this question













share|cite|improve this question











share|cite|improve this question




share|cite|improve this question










asked Nov 21 '18 at 5:28









Red Floyd

273314




273314












  • Have you learned about Taylor Series?
    – JavaMan
    Nov 21 '18 at 5:38










  • Yes, I have. Yea, maybe we can use taylor series to find the expansion. How do we show it converges? Also, is my statement correct about convergence about a different point in the interval?
    – Red Floyd
    Nov 21 '18 at 6:22






  • 1




    To show a Taylor Series converges, you have to show that $R_n(x) = sum_{k=n+1}^{infty} frac{f^{(k)}(a) (x-a)^k}{k!} to 0$. By Lagrange's Thm $R_n(x) = frac{f^{(n+1)}(c) (x-a)^{n+1}}{(n+1)!}$ for some $c$ between $a$ and $x$. Now, the $n$th derivative is something like $f(x) leq C x^{-n-2}$. Hence, $f^{(n)}(x) to 0$ as $n to infty$. In particular $f^{(n+1)}(c)$ is bounded. Finally, $(x-a)^{n+1}$ is polynomial, and factorials go to zero faster than any polynomial (this follows since $x^n/n! to 0$ as $n to infty$), which is demonstrated by convergence of Taylor series for $e^x$.
    – JavaMan
    Nov 22 '18 at 6:21


















  • Have you learned about Taylor Series?
    – JavaMan
    Nov 21 '18 at 5:38










  • Yes, I have. Yea, maybe we can use taylor series to find the expansion. How do we show it converges? Also, is my statement correct about convergence about a different point in the interval?
    – Red Floyd
    Nov 21 '18 at 6:22






  • 1




    To show a Taylor Series converges, you have to show that $R_n(x) = sum_{k=n+1}^{infty} frac{f^{(k)}(a) (x-a)^k}{k!} to 0$. By Lagrange's Thm $R_n(x) = frac{f^{(n+1)}(c) (x-a)^{n+1}}{(n+1)!}$ for some $c$ between $a$ and $x$. Now, the $n$th derivative is something like $f(x) leq C x^{-n-2}$. Hence, $f^{(n)}(x) to 0$ as $n to infty$. In particular $f^{(n+1)}(c)$ is bounded. Finally, $(x-a)^{n+1}$ is polynomial, and factorials go to zero faster than any polynomial (this follows since $x^n/n! to 0$ as $n to infty$), which is demonstrated by convergence of Taylor series for $e^x$.
    – JavaMan
    Nov 22 '18 at 6:21
















Have you learned about Taylor Series?
– JavaMan
Nov 21 '18 at 5:38




Have you learned about Taylor Series?
– JavaMan
Nov 21 '18 at 5:38












Yes, I have. Yea, maybe we can use taylor series to find the expansion. How do we show it converges? Also, is my statement correct about convergence about a different point in the interval?
– Red Floyd
Nov 21 '18 at 6:22




Yes, I have. Yea, maybe we can use taylor series to find the expansion. How do we show it converges? Also, is my statement correct about convergence about a different point in the interval?
– Red Floyd
Nov 21 '18 at 6:22




1




1




To show a Taylor Series converges, you have to show that $R_n(x) = sum_{k=n+1}^{infty} frac{f^{(k)}(a) (x-a)^k}{k!} to 0$. By Lagrange's Thm $R_n(x) = frac{f^{(n+1)}(c) (x-a)^{n+1}}{(n+1)!}$ for some $c$ between $a$ and $x$. Now, the $n$th derivative is something like $f(x) leq C x^{-n-2}$. Hence, $f^{(n)}(x) to 0$ as $n to infty$. In particular $f^{(n+1)}(c)$ is bounded. Finally, $(x-a)^{n+1}$ is polynomial, and factorials go to zero faster than any polynomial (this follows since $x^n/n! to 0$ as $n to infty$), which is demonstrated by convergence of Taylor series for $e^x$.
– JavaMan
Nov 22 '18 at 6:21




To show a Taylor Series converges, you have to show that $R_n(x) = sum_{k=n+1}^{infty} frac{f^{(k)}(a) (x-a)^k}{k!} to 0$. By Lagrange's Thm $R_n(x) = frac{f^{(n+1)}(c) (x-a)^{n+1}}{(n+1)!}$ for some $c$ between $a$ and $x$. Now, the $n$th derivative is something like $f(x) leq C x^{-n-2}$. Hence, $f^{(n)}(x) to 0$ as $n to infty$. In particular $f^{(n+1)}(c)$ is bounded. Finally, $(x-a)^{n+1}$ is polynomial, and factorials go to zero faster than any polynomial (this follows since $x^n/n! to 0$ as $n to infty$), which is demonstrated by convergence of Taylor series for $e^x$.
– JavaMan
Nov 22 '18 at 6:21










1 Answer
1






active

oldest

votes


















-2














Try just centering a geometric series around a different point. For example, take $$frac{1}{1-(-(x-x_0)^2))}$$



If this does have power series that converges, what would it look like? This would mean that you can get a radius 1 of converge around any point that you desire.






share|cite|improve this answer

















  • 1




    Please check your solution again.
    – Jacky Chong
    Nov 21 '18 at 6:39










  • unless I'm mistaken, we should get the series $sum_{i=1}^{infty} (-1)^i (x-x_0)^{2i}$ which converges whenever $|x-x_0|<1$?
    – Alex
    Nov 21 '18 at 6:43








  • 2




    I think the OP wants to consider $(1+x^2)^{-1}$.
    – Jacky Chong
    Nov 21 '18 at 6:46











Your Answer





StackExchange.ifUsing("editor", function () {
return StackExchange.using("mathjaxEditing", function () {
StackExchange.MarkdownEditor.creationCallbacks.add(function (editor, postfix) {
StackExchange.mathjaxEditing.prepareWmdForMathJax(editor, postfix, [["$", "$"], ["\\(","\\)"]]);
});
});
}, "mathjax-editing");

StackExchange.ready(function() {
var channelOptions = {
tags: "".split(" "),
id: "69"
};
initTagRenderer("".split(" "), "".split(" "), channelOptions);

StackExchange.using("externalEditor", function() {
// Have to fire editor after snippets, if snippets enabled
if (StackExchange.settings.snippets.snippetsEnabled) {
StackExchange.using("snippets", function() {
createEditor();
});
}
else {
createEditor();
}
});

function createEditor() {
StackExchange.prepareEditor({
heartbeatType: 'answer',
autoActivateHeartbeat: false,
convertImagesToLinks: true,
noModals: true,
showLowRepImageUploadWarning: true,
reputationToPostImages: 10,
bindNavPrevention: true,
postfix: "",
imageUploader: {
brandingHtml: "Powered by u003ca class="icon-imgur-white" href="https://imgur.com/"u003eu003c/au003e",
contentPolicyHtml: "User contributions licensed under u003ca href="https://creativecommons.org/licenses/by-sa/3.0/"u003ecc by-sa 3.0 with attribution requiredu003c/au003e u003ca href="https://stackoverflow.com/legal/content-policy"u003e(content policy)u003c/au003e",
allowUrls: true
},
noCode: true, onDemand: true,
discardSelector: ".discard-answer"
,immediatelyShowMarkdownHelp:true
});


}
});














draft saved

draft discarded


















StackExchange.ready(
function () {
StackExchange.openid.initPostLogin('.new-post-login', 'https%3a%2f%2fmath.stackexchange.com%2fquestions%2f3007301%2fshowing-given-function-is-real-analytic-on-r%23new-answer', 'question_page');
}
);

Post as a guest















Required, but never shown

























1 Answer
1






active

oldest

votes








1 Answer
1






active

oldest

votes









active

oldest

votes






active

oldest

votes









-2














Try just centering a geometric series around a different point. For example, take $$frac{1}{1-(-(x-x_0)^2))}$$



If this does have power series that converges, what would it look like? This would mean that you can get a radius 1 of converge around any point that you desire.






share|cite|improve this answer

















  • 1




    Please check your solution again.
    – Jacky Chong
    Nov 21 '18 at 6:39










  • unless I'm mistaken, we should get the series $sum_{i=1}^{infty} (-1)^i (x-x_0)^{2i}$ which converges whenever $|x-x_0|<1$?
    – Alex
    Nov 21 '18 at 6:43








  • 2




    I think the OP wants to consider $(1+x^2)^{-1}$.
    – Jacky Chong
    Nov 21 '18 at 6:46
















-2














Try just centering a geometric series around a different point. For example, take $$frac{1}{1-(-(x-x_0)^2))}$$



If this does have power series that converges, what would it look like? This would mean that you can get a radius 1 of converge around any point that you desire.






share|cite|improve this answer

















  • 1




    Please check your solution again.
    – Jacky Chong
    Nov 21 '18 at 6:39










  • unless I'm mistaken, we should get the series $sum_{i=1}^{infty} (-1)^i (x-x_0)^{2i}$ which converges whenever $|x-x_0|<1$?
    – Alex
    Nov 21 '18 at 6:43








  • 2




    I think the OP wants to consider $(1+x^2)^{-1}$.
    – Jacky Chong
    Nov 21 '18 at 6:46














-2












-2








-2






Try just centering a geometric series around a different point. For example, take $$frac{1}{1-(-(x-x_0)^2))}$$



If this does have power series that converges, what would it look like? This would mean that you can get a radius 1 of converge around any point that you desire.






share|cite|improve this answer












Try just centering a geometric series around a different point. For example, take $$frac{1}{1-(-(x-x_0)^2))}$$



If this does have power series that converges, what would it look like? This would mean that you can get a radius 1 of converge around any point that you desire.







share|cite|improve this answer












share|cite|improve this answer



share|cite|improve this answer










answered Nov 21 '18 at 6:34









Alex

1323




1323








  • 1




    Please check your solution again.
    – Jacky Chong
    Nov 21 '18 at 6:39










  • unless I'm mistaken, we should get the series $sum_{i=1}^{infty} (-1)^i (x-x_0)^{2i}$ which converges whenever $|x-x_0|<1$?
    – Alex
    Nov 21 '18 at 6:43








  • 2




    I think the OP wants to consider $(1+x^2)^{-1}$.
    – Jacky Chong
    Nov 21 '18 at 6:46














  • 1




    Please check your solution again.
    – Jacky Chong
    Nov 21 '18 at 6:39










  • unless I'm mistaken, we should get the series $sum_{i=1}^{infty} (-1)^i (x-x_0)^{2i}$ which converges whenever $|x-x_0|<1$?
    – Alex
    Nov 21 '18 at 6:43








  • 2




    I think the OP wants to consider $(1+x^2)^{-1}$.
    – Jacky Chong
    Nov 21 '18 at 6:46








1




1




Please check your solution again.
– Jacky Chong
Nov 21 '18 at 6:39




Please check your solution again.
– Jacky Chong
Nov 21 '18 at 6:39












unless I'm mistaken, we should get the series $sum_{i=1}^{infty} (-1)^i (x-x_0)^{2i}$ which converges whenever $|x-x_0|<1$?
– Alex
Nov 21 '18 at 6:43






unless I'm mistaken, we should get the series $sum_{i=1}^{infty} (-1)^i (x-x_0)^{2i}$ which converges whenever $|x-x_0|<1$?
– Alex
Nov 21 '18 at 6:43






2




2




I think the OP wants to consider $(1+x^2)^{-1}$.
– Jacky Chong
Nov 21 '18 at 6:46




I think the OP wants to consider $(1+x^2)^{-1}$.
– Jacky Chong
Nov 21 '18 at 6:46


















draft saved

draft discarded




















































Thanks for contributing an answer to Mathematics Stack Exchange!


  • Please be sure to answer the question. Provide details and share your research!

But avoid



  • Asking for help, clarification, or responding to other answers.

  • Making statements based on opinion; back them up with references or personal experience.


Use MathJax to format equations. MathJax reference.


To learn more, see our tips on writing great answers.





Some of your past answers have not been well-received, and you're in danger of being blocked from answering.


Please pay close attention to the following guidance:


  • Please be sure to answer the question. Provide details and share your research!

But avoid



  • Asking for help, clarification, or responding to other answers.

  • Making statements based on opinion; back them up with references or personal experience.


To learn more, see our tips on writing great answers.




draft saved


draft discarded














StackExchange.ready(
function () {
StackExchange.openid.initPostLogin('.new-post-login', 'https%3a%2f%2fmath.stackexchange.com%2fquestions%2f3007301%2fshowing-given-function-is-real-analytic-on-r%23new-answer', 'question_page');
}
);

Post as a guest















Required, but never shown





















































Required, but never shown














Required, but never shown












Required, but never shown







Required, but never shown

































Required, but never shown














Required, but never shown












Required, but never shown







Required, but never shown







Popular posts from this blog

Can a sorcerer learn a 5th-level spell early by creating spell slots using the Font of Magic feature?

Does disintegrating a polymorphed enemy still kill it after the 2018 errata?

A Topological Invariant for $pi_3(U(n))$